Prove that the angular momentum operator is hermitian

In summary, the proof for the hermitian nature of the angular momentum operator involves taking the Hermitian conjugate of the operator, resulting in a reversal of the operator order and a minus sign when using vector notation. This is demonstrated in the derivation process, but a flaw arises when the arguments of the cross product are also reversed, which is not necessary.
  • #1
Septim
167
6
Greetings,

My task is to prove that the angular momentum operator is hermitian. I started out as follows:

[itex]\vec{L}=\vec{r}\times\vec{p}[/itex]

Where the above quantities are vector operators. Taking the hermitian conjugate yields

[itex]\vec{L''}=\vec{p''}\times\vec{r''}[/itex]

Here I have used double quotes to represent that the hermitian conjugate of the corresponding quantity.

[itex]\vec{L''}=\vec{p}\times\vec{r}[/itex]

Here the fact that the momentum and position are hermitian operators were used. However
[itex]\vec{L''}=\vec{p}\times\vec{r}=-\vec{r}\times\vec{p}=-\vec{L}[[/itex]

There has to be a flaw somewhere but I was not able to catch it, though I was able to prove that the angular momentum operator is hermitian when inspected component by component. I am yet to understand the error in the above derivation. Any help is appreciated.

Thanks in advance
 
Physics news on Phys.org
  • #2
L′′ = p′′ × r′′

The Hermitian conjugate reverses the operator order, but you've also reversed the order of the arguments to the cross product, which should bring in a minus sign.
 
  • #3
Thanks for the reply. So as far as I am concerned you tell me that reversing the arguments of the cross product is superfluous. But how can I denote that I have reversed the orders of the operators without doing that? My point is to demonstrate that I have reversed the order of operators in vector notation.
 
  • #4
Septim said:
Thanks for the reply. So as far as I am concerned you tell me that reversing the arguments of the cross product is superfluous. But how can I denote that I have reversed the orders of the operators without doing that? My point is to demonstrate that I have reversed the order of operators in vector notation.

In general, if [itex]\vec{A}[/itex] and [itex]\vec{B}[/itex] are operators, then [itex](\vec{A} \times \vec{B})^\dagger = - \vec{B}^\dagger \times \vec{A}^\dagger[/itex]
 

1. What is the definition of a Hermitian operator?

A Hermitian operator is a mathematical operator that satisfies the condition of being equal to its own conjugate transpose. In other words, it is equal to its own complex conjugate when transposed.

2. How do you prove that the angular momentum operator is Hermitian?

The proof involves showing that the angular momentum operator, represented by L, satisfies the condition L† = L, where L† is the conjugate transpose of L. This can be done by using the properties of complex conjugates and the commutator relations of the angular momentum operators.

3. Why is it important for an operator to be Hermitian?

Hermitian operators have several important properties that make them useful in quantum mechanics. They have real eigenvalues, which correspond to physically observable quantities, and their eigenstates form a complete orthonormal basis. Additionally, Hermitian operators are self-adjoint, meaning they are their own inverse, which simplifies calculations and allows for the use of the spectral theorem.

4. Can you provide an example of an angular momentum operator that is not Hermitian?

Yes, the operator for the z component of angular momentum, Lz, is not Hermitian. This can be shown by calculating its conjugate transpose, Lz†, which is equal to -Lz. Therefore, Lz† is not equal to Lz, violating the condition for a Hermitian operator.

5. How does the Hermiticity of the angular momentum operator relate to the conservation of angular momentum in quantum systems?

The Hermiticity of the angular momentum operator is directly related to the conservation of angular momentum in quantum systems. This is because the operator is a mathematical representation of the physical property of angular momentum, and its Hermiticity ensures that this property is conserved in quantum mechanical systems.

Similar threads

  • Quantum Physics
Replies
7
Views
985
Replies
13
Views
172
Replies
10
Views
1K
Replies
6
Views
863
Replies
7
Views
1K
  • Quantum Physics
Replies
1
Views
901
Replies
14
Views
1K
  • Quantum Physics
Replies
4
Views
742
Replies
2
Views
729
Replies
3
Views
782
Back
Top